Difference between revisions of "2018 AMC 8 Problems/Problem 21"

(Video Solutions)
Line 13: Line 13:
 
==Solution 3==
 
==Solution 3==
 
By the [[Chinese Remainder Theorem]], we have that all solutions are in the form <math>x=198k+194</math> where <math>k\in \mathbb{Z}.</math> Counting the number of values, we get <math>\boxed{\textbf{(E) }5}.</math>
 
By the [[Chinese Remainder Theorem]], we have that all solutions are in the form <math>x=198k+194</math> where <math>k\in \mathbb{Z}.</math> Counting the number of values, we get <math>\boxed{\textbf{(E) }5}.</math>
 +
 +
==Solution 4==
 +
We can use modular arithmetic. Set up the equations: <math>x &\equiv 2 \mod 6,</math> <math>x &\equiv 5 \mod 9,</math> and <math>x &\equiv 7 \mod 11.</math> These equations can also be written as <math>x+4 &\equiv 0 \mod 6,</math> <math>x+4 &\equiv 0 \mod 9,</math> and <math>x+4 &\equiv 0 \mod 11.</math> Since <math>x+4</math> is congruent to numbers <math>6, 9,</math> and <math>11,</math> then it must also be congruent to their LCM. Thus, <math>x+4 &\equiv 0 \mod 198,</math> since 198 is the LCM of <math>6, 9,</math> and <math>11.</math> Since these numbers have to be three digit, they can only be <math>194, 392, 590, 788,</math> and <math>986.</math> This gives us the answer of <math>\boxed{\textbf{(E) }5}.</math>
  
 
~mathboy282
 
~mathboy282

Revision as of 15:58, 17 December 2022

Problem

How many positive three-digit integers have a remainder of 2 when divided by 6, a remainder of 5 when divided by 9, and a remainder of 7 when divided by 11?

$\textbf{(A) }1\qquad\textbf{(B) }2\qquad\textbf{(C) }3\qquad\textbf{(D) }4\qquad \textbf{(E) }5$

Solution 1

Looking at the values, we notice that $11-7=4$, $9-5=4$ and $6-2=4$. This means we are looking for a value that is four less than a multiple of $11$, $9$, and $6$. The least common multiple of these numbers is $11\cdot3^{2}\cdot2=198$, so the numbers that fulfill this can be written as $198k-4$, where $k$ is a positive integer. This value is only a three digit integer when $k$ is $1, 2, 3, 4$ or $5$, which gives $194, 392, 590, 788,$ and $986$ respectively. Thus we have $5$ values, so our answer is $\boxed{\textbf{(E) }5}$

Solution 2

Let us create the equations: $6x+2 = 9y+5 = 11z+7$, and we know $100 \leq 11z+7 <1000$, it gives us $9 \leq z \leq 90$, which is the range of the value of z. Because of $6x+2=11z+7$, then $6x=11z+5=6z+5(z+1)$, so $(z+1)$ must be a mutiple of 6. Because of $9y+5=11z+7$, then $9y=11z+2=9z+2(z+1)$, so $(z+1)$ must also be a mutiple of $9$. Hence, the value of $(z+1)$ must be a common multiple of $6$ and $9$, which means multiples of $18(LCM \text{ of }\ 6, 9)$. So let's say $z+1 = 18p$, then $9 \leq z = 18p-1 \leq 90$, so $1 \leq p \leq 91/18\ or \ 1 \leq p \leq 5$. Thus the answer is $\boxed{\textbf{(E) }5}$ ~LarryFlora

Solution 3

By the Chinese Remainder Theorem, we have that all solutions are in the form $x=198k+194$ where $k\in \mathbb{Z}.$ Counting the number of values, we get $\boxed{\textbf{(E) }5}.$

Solution 4

We can use modular arithmetic. Set up the equations: $x &\equiv 2 \mod 6,$ (Error compiling LaTeX. Unknown error_msg) $x &\equiv 5 \mod 9,$ (Error compiling LaTeX. Unknown error_msg) and $x &\equiv 7 \mod 11.$ (Error compiling LaTeX. Unknown error_msg) These equations can also be written as $x+4 &\equiv 0 \mod 6,$ (Error compiling LaTeX. Unknown error_msg) $x+4 &\equiv 0 \mod 9,$ (Error compiling LaTeX. Unknown error_msg) and $x+4 &\equiv 0 \mod 11.$ (Error compiling LaTeX. Unknown error_msg) Since $x+4$ is congruent to numbers $6, 9,$ and $11,$ then it must also be congruent to their LCM. Thus, $x+4 &\equiv 0 \mod 198,$ (Error compiling LaTeX. Unknown error_msg) since 198 is the LCM of $6, 9,$ and $11.$ Since these numbers have to be three digit, they can only be $194, 392, 590, 788,$ and $986.$ This gives us the answer of $\boxed{\textbf{(E) }5}.$

~mathboy282

Video Solutions

https://youtu.be/CPQpkpnEuIc - Happytwin

https://youtu.be/PTwMDbsz2xI

https://youtu.be/7an5wU9Q5hk?t=939 - pi_is_3.14

https://youtu.be/hoCdk8AC-0c

~savannahsolver

https://www.youtube.com/watch?v=PjYwbGm_2aM

See Also

2018 AMC 8 (ProblemsAnswer KeyResources)
Preceded by
Problem 20
Followed by
Problem 22
1 2 3 4 5 6 7 8 9 10 11 12 13 14 15 16 17 18 19 20 21 22 23 24 25
All AJHSME/AMC 8 Problems and Solutions

The problems on this page are copyrighted by the Mathematical Association of America's American Mathematics Competitions. AMC logo.png